The Stacks project

Lemma 20.7.3. Let $f : X \to Y$ be a morphism of ringed spaces. Let $\mathcal{F}$ be a $\mathcal{O}_ X$-module. The sheaves $R^ if_*\mathcal{F}$ are the sheaves associated to the presheaves

\[ V \longmapsto H^ i(f^{-1}(V), \mathcal{F}) \]

with restriction mappings as in Equation (20.7.1.1). There is a similar statement for $R^ if_*$ applied to a bounded below complex $\mathcal{F}^\bullet $.

Proof. Let $\mathcal{F} \to \mathcal{I}^\bullet $ be an injective resolution. Then $R^ if_*\mathcal{F}$ is by definition the $i$th cohomology sheaf of the complex

\[ f_*\mathcal{I}^0 \to f_*\mathcal{I}^1 \to f_*\mathcal{I}^2 \to \ldots \]

By definition of the abelian category structure on $\mathcal{O}_ Y$-modules this cohomology sheaf is the sheaf associated to the presheaf

\[ V \longmapsto \frac{\mathop{\mathrm{Ker}}(f_*\mathcal{I}^ i(V) \to f_*\mathcal{I}^{i + 1}(V))}{\mathop{\mathrm{Im}}(f_*\mathcal{I}^{i - 1}(V) \to f_*\mathcal{I}^ i(V))} \]

and this is obviously equal to

\[ \frac{\mathop{\mathrm{Ker}}(\mathcal{I}^ i(f^{-1}(V)) \to \mathcal{I}^{i + 1}(f^{-1}(V)))}{\mathop{\mathrm{Im}}(\mathcal{I}^{i - 1}(f^{-1}(V)) \to \mathcal{I}^ i(f^{-1}(V)))} \]

which is equal to $H^ i(f^{-1}(V), \mathcal{F})$ and we win. $\square$


Comments (4)

Comment #1811 by Keenan Kidwell on

Is there something non-trivial being used here about sheafification and quotients? The second displayed formula is the presheaf quotient of the kernel of by the subpresheaf given by the presheaf image of . But by definition, the cohomology sheaf is the sheaf associated to the presheaf quotient of the aforementioned kernel by the sheaf-theoretic image of . That these are the same follows by e.g. exactness of sheafification, right? Or is there something simpler that I'm missing?

Comment #1829 by on

Nope, you are not missing anything. The problem with statements such as these is really that once you understand everything well enough, it actually becomes confusing when you try to spell out what is going on. At least that is my take on it.

Comment #4643 by Andy on

Adding to the discussion here, I think another point of view is that pullback of sheaves is forgetting it's a sheaf, pulling back, and sheafifying. And the second and third functor do not contribute in the derived setting, hence the statement. I'm not claiming this is any easier than the proof given of course.

Comment #4644 by Andy on

Sorry I meant to say pushforward, but still should be exact in the presheaf setting.


Post a comment

Your email address will not be published. Required fields are marked.

In your comment you can use Markdown and LaTeX style mathematics (enclose it like $\pi$). A preview option is available if you wish to see how it works out (just click on the eye in the toolbar).

Unfortunately JavaScript is disabled in your browser, so the comment preview function will not work.

All contributions are licensed under the GNU Free Documentation License.




In order to prevent bots from posting comments, we would like you to prove that you are human. You can do this by filling in the name of the current tag in the following input field. As a reminder, this is tag 01E4. Beware of the difference between the letter 'O' and the digit '0'.